// Numbas version: exam_results_page_options {"name": "Simon's copy of Particle in equilibrium on an incline.", "extensions": [], "custom_part_types": [], "resources": [["question-resources/statics2.png", "/srv/numbas/media/question-resources/statics2.png"], ["question-resources/statics3.png", "/srv/numbas/media/question-resources/statics3.png"], ["question-resources/Particle_in_equilibrium_on_a_plane.png", "/srv/numbas/media/question-resources/Particle_in_equilibrium_on_a_plane.png"], ["question-resources/Particle_in_equilibrium_on_a_plane_solution_gxmuyuG.png", "/srv/numbas/media/question-resources/Particle_in_equilibrium_on_a_plane_solution_gxmuyuG.png"]], "navigation": {"allowregen": true, "showfrontpage": false, "preventleave": false, "typeendtoleave": false}, "question_groups": [{"pickingStrategy": "all-ordered", "questions": [{"name": "Simon's copy of Particle in equilibrium on an incline.", "preamble": {"css": "", "js": ""}, "variable_groups": [], "metadata": {"licence": "Creative Commons Attribution 4.0 International", "description": "

A particle is in equilibrium on an incline. Four forces are acting on it. You're given the angle of the incline and three forces. Resolve the forces to find the angle and magnitude of the other force.

"}, "advice": "

We can draw our $x$ and $y$ axis as being parallel and perpendicular to the particle, as shown in the diagram below and then resolve the forces.

\n

\n

Resolving in the $x$ direction gives equation (1)

\n

\\begin{align}
F_3 + F_4 \\cos(90^{\\circ} - \\theta_1) - F_2 \\cos \\theta_2 & = 0,\\\\[0.5em]
F_2 \\cos \\theta_2 & = F_3 + F_4 \\sin \\theta_1, \\\\
&= \\var{F3} + \\var{F4} \\sin \\var{theta1}^{\\circ}. && (1) 
\\end{align}

\n

Resolving in the $y$ direction gives equation (2)

\n

\\begin{align}
F_1 - F_4 \\cos \\theta_1 + F_2 \\sin \\theta_2 & = 0,\\\\[0.5em]
F_2 \\sin \\theta_2 & = F_4 \\cos \\theta_1 - F_1, \\\\
&= \\var{F4} \\cos \\var{theta1}^{\\circ} - \\var{F1}. && (2) 
\\end{align}

\n

Dividing equation (2) by equation (1) gives (to 3 s.f.)

\n

\\begin{align}
\\frac{F_2 \\sin \\theta_2}{F_2 \\cos \\theta_2} & = \\frac{\\var{F4} \\cos \\var{theta1}^{\\circ} - \\var{F1}}{\\var{F3} + \\var{F4} \\sin \\var{theta1}^{\\circ}}, \\\\[0.5em]
\\tan \\theta_2 & = \\frac{\\var{F4} \\cos \\var{theta1}^{\\circ} - \\var{F1}}{\\var{F3} + \\var{F4} \\sin \\var{theta1}^{\\circ}}, \\\\[0.5em]
\\theta_2 & = \\arctan \\left(\\frac{\\var{F4} \\cos \\var{theta1}^{\\circ} - \\var{F1}}{\\var{F3} + \\var{F4} \\sin \\var{theta1}^{\\circ}}\\right), \\\\[0.5em]
& = \\var{thetadp}^{\\circ}.
\\end{align} 

\n

We can then substitute the value for $\\theta_2$ in either equation (1) or (2) to find $F_2$.

\n

Using equation (1), we get

\n

\\begin{align} 
F_2 \\cos \\theta_2 & = \\var{F3} + \\var{F4} \\sin \\var{theta1}^{\\circ}, \\\\[0.5em]
F_2 & = \\frac{\\var{F3} + \\var{F4} \\sin \\var{theta1}^{\\circ}}{\\cos \\var{thetadp}^{\\circ}}, \\\\[0.5em]
& = \\var{precround( (F3 + F4*sin(radians(theta1)))/(cos(radians(thetadp))),3)} \\ \\mathrm{N}.
\\end{align}

", "ungrouped_variables": ["F1", "F3", "F4", "theta1", "tantheta2", "theta2", "F2", "thetadp", "F2check", "F2sig"], "variablesTest": {"condition": "F2>0", "maxRuns": 100}, "tags": [], "statement": "

The diagram below shows a particle in equilibrium under the action of four forces.

\n

\n

You are given the following information:

\n", "rulesets": {}, "functions": {}, "parts": [{"showCorrectAnswer": true, "showFeedbackIcon": true, "extendBaseMarkingAlgorithm": true, "prompt": "

By resolving forces in components find, to 3 d.p., the unknown angle $\\theta_2$.

\n

$\\theta_2 = $ [[0]]

", "type": "gapfill", "unitTests": [], "variableReplacementStrategy": "originalfirst", "variableReplacements": [], "scripts": {}, "sortAnswers": false, "marks": 0, "gaps": [{"minValue": "theta2", "showCorrectAnswer": true, "precisionType": "dp", "mustBeReducedPC": 0, "precision": "3", "maxValue": "theta2", "allowFractions": false, "strictPrecision": false, "unitTests": [], "variableReplacements": [], "precisionMessage": "

You have not given your answer to the correct precision.

", "marks": 1, "correctAnswerFraction": false, "precisionPartialCredit": 0, "showFeedbackIcon": true, "extendBaseMarkingAlgorithm": true, "scripts": {}, "mustBeReduced": false, "correctAnswerStyle": "plain", "type": "numberentry", "notationStyles": ["plain", "en", "si-en"], "showPrecisionHint": false, "variableReplacementStrategy": "originalfirst", "customMarkingAlgorithm": ""}], "customMarkingAlgorithm": ""}, {"showCorrectAnswer": true, "showFeedbackIcon": true, "extendBaseMarkingAlgorithm": true, "prompt": "

Using your answer from part a) find the magnitude of the force $F_2$, in Newtons to 3 d.p.

\n

$F_2 = $ [[0]]

", "type": "gapfill", "unitTests": [], "variableReplacementStrategy": "originalfirst", "variableReplacements": [], "scripts": {}, "sortAnswers": false, "marks": 0, "gaps": [{"minValue": "F2", "showCorrectAnswer": true, "precisionType": "dp", "mustBeReducedPC": 0, "precision": "3", "maxValue": "F2", "allowFractions": false, "strictPrecision": false, "unitTests": [], "variableReplacements": [], "precisionMessage": "

You have not given your answer to the correct precision.

", "marks": 1, "correctAnswerFraction": false, "precisionPartialCredit": 0, "showFeedbackIcon": true, "extendBaseMarkingAlgorithm": true, "scripts": {}, "mustBeReduced": false, "correctAnswerStyle": "plain", "type": "numberentry", "notationStyles": ["plain", "en", "si-en"], "showPrecisionHint": false, "variableReplacementStrategy": "originalfirst", "customMarkingAlgorithm": ""}], "customMarkingAlgorithm": ""}], "extensions": [], "variables": {"theta1": {"name": "theta1", "description": "", "group": "Ungrouped variables", "templateType": "randrange", "definition": "random(10..40#1)"}, "F2sig": {"name": "F2sig", "description": "", "group": "Ungrouped variables", "templateType": "anything", "definition": "siground(F2,3)"}, "F4": {"name": "F4", "description": "", "group": "Ungrouped variables", "templateType": "randrange", "definition": "random(5.1..8#0.1)"}, "F2": {"name": "F2", "description": "

uses theta2 to 3dp

", "group": "Ungrouped variables", "templateType": "anything", "definition": "(F3 + F4*sin(radians(theta1)))/(cos(radians(thetadp)))"}, "F3": {"name": "F3", "description": "", "group": "Ungrouped variables", "templateType": "randrange", "definition": "random(0.5..2.8#0.1)"}, "thetadp": {"name": "thetadp", "description": "

theta2 to 3d.p. (part a solution will be used in part b - which will allow for rounding errors)

", "group": "Ungrouped variables", "templateType": "anything", "definition": "precround(theta2,3)"}, "tantheta2": {"name": "tantheta2", "description": "", "group": "Ungrouped variables", "templateType": "anything", "definition": "(F4*cos(radians(theta1))-F1)/(F3+F4*cos(radians(90-theta1)))"}, "F1": {"name": "F1", "description": "", "group": "Ungrouped variables", "templateType": "randrange", "definition": "random(3..4#0.1)"}, "theta2": {"name": "theta2", "description": "", "group": "Ungrouped variables", "templateType": "anything", "definition": "degrees(arctan(tantheta2))"}, "F2check": {"name": "F2check", "description": "

uses theta2

", "group": "Ungrouped variables", "templateType": "anything", "definition": "(F3 + F4*sin(radians(theta1)))/(cos(radians(theta2)))"}}, "type": "question", "contributors": [{"name": "Amy Chadwick", "profile_url": "https://numbas.mathcentre.ac.uk/accounts/profile/505/"}, {"name": "Simon Thomas", "profile_url": "https://numbas.mathcentre.ac.uk/accounts/profile/3148/"}]}]}], "contributors": [{"name": "Amy Chadwick", "profile_url": "https://numbas.mathcentre.ac.uk/accounts/profile/505/"}, {"name": "Simon Thomas", "profile_url": "https://numbas.mathcentre.ac.uk/accounts/profile/3148/"}]}